Search found 84 matches


(1/2)^x>(1/1000)

2^x>1000


2,4,8,16,32,64,128,256,512,1024

x=10

I don't know if there are any other ways to solve it, but this is the formula way.

Please correct me if I made a mistake.

by EricLien9122

Tue Nov 18, 2008 9:20 am
Forum: Problem Solving
Topic: Probability
Replies: 6
Views: 1378

It's hard to predict the out come of divisibility, because the outcome can be a fraction regardless the even or odd rules. Q: set x has n number of int. is mean even? Statement 1: n is even mean can be fraction, odd or even insufficient Statement 2: all int. in x are even mean can be fraction or eve...

by EricLien9122

Tue Nov 18, 2008 7:14 am
Forum: Data Sufficiency
Topic: Mean Even
Replies: 9
Views: 2205

Remember X and Y are both greater than 0, so -1 is not a possible answer choice :D

by EricLien9122

Tue Nov 18, 2008 6:51 am
Forum: Data Sufficiency
Topic: number prop / vic from gmat2 - corrected!
Replies: 6
Views: 1606

2. Rain,Rain,Rain, N.Rain,N.Rain

probability of exactly 3 days of rain= (# of ways with 3 days of rain)/(total number of possibility of a raining day in 5 days)

(5!)/(2!3!)/(5!)=1/(2*2*3)=1/12

I think this is the right answer, please correct me if i made a mistake.

by EricLien9122

Mon Nov 17, 2008 9:12 pm
Forum: Problem Solving
Topic: simple but tricky
Replies: 10
Views: 6142

Q: P(w)+P(E)=?

1. P(w)*P(E)=0

Insufficient, b/c we can't manipulate the equation into P(W)+P(E)

2. P(W)-P(E)=0.2

Insufficient, b/c we can't manipulate the question in to P(W)+P(E)

1+2:

Still insufficient, b/c we can't manipulate the equation into P(E)+P(E).

by EricLien9122

Sun Nov 16, 2008 1:06 pm
Forum: Data Sufficiency
Topic: gmat prep ds
Replies: 7
Views: 2053

In must be true questions, try to disprove the answer choice. A. x>2, if x=-1, then x is not greater than 2. B. x>-7, smallest x=-1.999 or largest x=6.9999, which both are greater than -7. C. x<2, if x=6, then x is not less than 2. D.-7<x<2, if x=5, then x is not in that range. I hope this help, ple...

by EricLien9122

Sun Nov 16, 2008 12:38 am
Forum: Problem Solving
Topic: PS (Must be True) Explanation
Replies: 3
Views: 1389

I think the big problem with this sentence is "they" refers to lawyers while should refers to judge. we can rule out A and C. D: "them refers to lawyers while should refer to patent holder. B: act (purual) should be acts (singular). as for E, I don't see a subjunctive mood. It's only ...

by EricLien9122

Sun Nov 16, 2008 12:26 am
Forum: Sentence Correction
Topic: Manhattan CAT Q...problem with subjunctive mood
Replies: 9
Views: 2687

just curious, what's the source of this question? I always thought the actual GMAT would have easier number for weighted average calculation.

by EricLien9122

Sat Nov 15, 2008 11:32 pm
Forum: Problem Solving
Topic: Question 33 - Help please :)
Replies: 6
Views: 2079

very good stuff Logic, thanks for sharing with us.

by EricLien9122

Sat Nov 15, 2008 11:15 pm
Forum: Reading Comprehension
Topic: Common GMAT passage structures
Replies: 3
Views: 3984

This is an experiment type question. The experiment usually assume their "data" or result is representative. The conclusion: A with classic: cried less, fewer ailments, and more weights than B with rock music. In answer E, does it matter if music were play at different time? The experiment...

by EricLien9122

Sat Nov 15, 2008 10:58 pm
Forum: Critical Reasoning
Topic: Music
Replies: 6
Views: 3441

B: 3
G: 4

(3x+5)/4x=1

x=5

5(4)=20

The trick is adding 5 to the boys in order to maintain a 1 to 1 ratio.

I hope this makes sense, please correct me if I made a mistake.

by EricLien9122

Fri Nov 14, 2008 1:14 pm
Forum: Problem Solving
Topic: Question 5 (nov.14th)
Replies: 2
Views: 1083

In my opinion, the answer is D.

I will go in to details if my answer is correct.

by EricLien9122

Fri Nov 14, 2008 1:03 pm
Forum: Data Sufficiency
Topic: Another Question
Replies: 7
Views: 1814

Is answer A?

A4=2(27)-x
A4=94-x

99=2(94-x)-x
99=108-2x-x
-9=-3x
x=3

by EricLien9122

Fri Nov 14, 2008 12:50 pm
Forum: Problem Solving
Topic: Question 2 (Nov.14)
Replies: 4
Views: 4480

Hey Ron, thanks for the amazing explanation, but I am still not sure if C is sufficient, because the neither statement stated that the "middle" number is equal to 1000. So...we can have -20,-10, 9999, 10000, 10001 median=9999 we also can have... -20,-10,1000,1001,1002 median=1000 Since we ...

by EricLien9122

Fri Nov 14, 2008 11:49 am
Forum: Data Sufficiency
Topic: Median
Replies: 17
Views: 3562

pass Not pass A: 20 10 B: 17 13 C: 15 15 So, we know at least 15 teachers failed the test. Since the question didn't tell us any relationship between A, B and C, and asked about how many at least passed (minimum value), we have to assume the worst possible situation. C failed 15, A and B failed 15 t...

by EricLien9122

Fri Nov 14, 2008 10:47 am
Forum: Problem Solving
Topic: Sets - confusing OA
Replies: 6
Views: 1670